Joint density of two vectors of multivariate normal random variables












1














If $bf{X}$ and $bf{Y}$ are dependent multivariate normal random variables, what is the joint density of $bf{X}$ and $bf{Y}$? Is it also multivariate normal?










share|cite|improve this question




















  • 2




    On the other way round: If you know that they are independent, then they are jointly multivariate normal. If you only know they are dependent, then they are not necessary to be jointly multivariate normal in general.
    – BGM
    Nov 21 '18 at 6:27
















1














If $bf{X}$ and $bf{Y}$ are dependent multivariate normal random variables, what is the joint density of $bf{X}$ and $bf{Y}$? Is it also multivariate normal?










share|cite|improve this question




















  • 2




    On the other way round: If you know that they are independent, then they are jointly multivariate normal. If you only know they are dependent, then they are not necessary to be jointly multivariate normal in general.
    – BGM
    Nov 21 '18 at 6:27














1












1








1







If $bf{X}$ and $bf{Y}$ are dependent multivariate normal random variables, what is the joint density of $bf{X}$ and $bf{Y}$? Is it also multivariate normal?










share|cite|improve this question















If $bf{X}$ and $bf{Y}$ are dependent multivariate normal random variables, what is the joint density of $bf{X}$ and $bf{Y}$? Is it also multivariate normal?







probability matrices normal-distribution






share|cite|improve this question















share|cite|improve this question













share|cite|improve this question




share|cite|improve this question








edited Nov 21 '18 at 10:43









p4sch

4,760217




4,760217










asked Nov 21 '18 at 5:27









shani

1208




1208








  • 2




    On the other way round: If you know that they are independent, then they are jointly multivariate normal. If you only know they are dependent, then they are not necessary to be jointly multivariate normal in general.
    – BGM
    Nov 21 '18 at 6:27














  • 2




    On the other way round: If you know that they are independent, then they are jointly multivariate normal. If you only know they are dependent, then they are not necessary to be jointly multivariate normal in general.
    – BGM
    Nov 21 '18 at 6:27








2




2




On the other way round: If you know that they are independent, then they are jointly multivariate normal. If you only know they are dependent, then they are not necessary to be jointly multivariate normal in general.
– BGM
Nov 21 '18 at 6:27




On the other way round: If you know that they are independent, then they are jointly multivariate normal. If you only know they are dependent, then they are not necessary to be jointly multivariate normal in general.
– BGM
Nov 21 '18 at 6:27










1 Answer
1






active

oldest

votes


















0














As already noted in the comments: In general, it is not true that $X$ and $Y$ have a jointly multivariate normal distribution. Here is one counterexample: Let $X sim mathcal{N}(0,1)$ and $Y$ independent of $X$ with $P(Y=1) =1/2$ and $P(Y=-1)=1/2$. Now set $Z=XY$.




  • Show that $Z$ has also normal distribution with mean $0$ and variance $1$, i.e. $Z sim mathcal{N}(0,1)$.


  • $X$ and $Z$ are uncorrelated.


  • $X$ and $Z$ are not independent. (For example $P( |X| le t, |Z| >t) =0$, but $P(|X| le t) P(|Z|>t) ne 0$ for $t>0$.)






share|cite|improve this answer





















    Your Answer





    StackExchange.ifUsing("editor", function () {
    return StackExchange.using("mathjaxEditing", function () {
    StackExchange.MarkdownEditor.creationCallbacks.add(function (editor, postfix) {
    StackExchange.mathjaxEditing.prepareWmdForMathJax(editor, postfix, [["$", "$"], ["\\(","\\)"]]);
    });
    });
    }, "mathjax-editing");

    StackExchange.ready(function() {
    var channelOptions = {
    tags: "".split(" "),
    id: "69"
    };
    initTagRenderer("".split(" "), "".split(" "), channelOptions);

    StackExchange.using("externalEditor", function() {
    // Have to fire editor after snippets, if snippets enabled
    if (StackExchange.settings.snippets.snippetsEnabled) {
    StackExchange.using("snippets", function() {
    createEditor();
    });
    }
    else {
    createEditor();
    }
    });

    function createEditor() {
    StackExchange.prepareEditor({
    heartbeatType: 'answer',
    autoActivateHeartbeat: false,
    convertImagesToLinks: true,
    noModals: true,
    showLowRepImageUploadWarning: true,
    reputationToPostImages: 10,
    bindNavPrevention: true,
    postfix: "",
    imageUploader: {
    brandingHtml: "Powered by u003ca class="icon-imgur-white" href="https://imgur.com/"u003eu003c/au003e",
    contentPolicyHtml: "User contributions licensed under u003ca href="https://creativecommons.org/licenses/by-sa/3.0/"u003ecc by-sa 3.0 with attribution requiredu003c/au003e u003ca href="https://stackoverflow.com/legal/content-policy"u003e(content policy)u003c/au003e",
    allowUrls: true
    },
    noCode: true, onDemand: true,
    discardSelector: ".discard-answer"
    ,immediatelyShowMarkdownHelp:true
    });


    }
    });














    draft saved

    draft discarded


















    StackExchange.ready(
    function () {
    StackExchange.openid.initPostLogin('.new-post-login', 'https%3a%2f%2fmath.stackexchange.com%2fquestions%2f3007300%2fjoint-density-of-two-vectors-of-multivariate-normal-random-variables%23new-answer', 'question_page');
    }
    );

    Post as a guest















    Required, but never shown

























    1 Answer
    1






    active

    oldest

    votes








    1 Answer
    1






    active

    oldest

    votes









    active

    oldest

    votes






    active

    oldest

    votes









    0














    As already noted in the comments: In general, it is not true that $X$ and $Y$ have a jointly multivariate normal distribution. Here is one counterexample: Let $X sim mathcal{N}(0,1)$ and $Y$ independent of $X$ with $P(Y=1) =1/2$ and $P(Y=-1)=1/2$. Now set $Z=XY$.




    • Show that $Z$ has also normal distribution with mean $0$ and variance $1$, i.e. $Z sim mathcal{N}(0,1)$.


    • $X$ and $Z$ are uncorrelated.


    • $X$ and $Z$ are not independent. (For example $P( |X| le t, |Z| >t) =0$, but $P(|X| le t) P(|Z|>t) ne 0$ for $t>0$.)






    share|cite|improve this answer


























      0














      As already noted in the comments: In general, it is not true that $X$ and $Y$ have a jointly multivariate normal distribution. Here is one counterexample: Let $X sim mathcal{N}(0,1)$ and $Y$ independent of $X$ with $P(Y=1) =1/2$ and $P(Y=-1)=1/2$. Now set $Z=XY$.




      • Show that $Z$ has also normal distribution with mean $0$ and variance $1$, i.e. $Z sim mathcal{N}(0,1)$.


      • $X$ and $Z$ are uncorrelated.


      • $X$ and $Z$ are not independent. (For example $P( |X| le t, |Z| >t) =0$, but $P(|X| le t) P(|Z|>t) ne 0$ for $t>0$.)






      share|cite|improve this answer
























        0












        0








        0






        As already noted in the comments: In general, it is not true that $X$ and $Y$ have a jointly multivariate normal distribution. Here is one counterexample: Let $X sim mathcal{N}(0,1)$ and $Y$ independent of $X$ with $P(Y=1) =1/2$ and $P(Y=-1)=1/2$. Now set $Z=XY$.




        • Show that $Z$ has also normal distribution with mean $0$ and variance $1$, i.e. $Z sim mathcal{N}(0,1)$.


        • $X$ and $Z$ are uncorrelated.


        • $X$ and $Z$ are not independent. (For example $P( |X| le t, |Z| >t) =0$, but $P(|X| le t) P(|Z|>t) ne 0$ for $t>0$.)






        share|cite|improve this answer












        As already noted in the comments: In general, it is not true that $X$ and $Y$ have a jointly multivariate normal distribution. Here is one counterexample: Let $X sim mathcal{N}(0,1)$ and $Y$ independent of $X$ with $P(Y=1) =1/2$ and $P(Y=-1)=1/2$. Now set $Z=XY$.




        • Show that $Z$ has also normal distribution with mean $0$ and variance $1$, i.e. $Z sim mathcal{N}(0,1)$.


        • $X$ and $Z$ are uncorrelated.


        • $X$ and $Z$ are not independent. (For example $P( |X| le t, |Z| >t) =0$, but $P(|X| le t) P(|Z|>t) ne 0$ for $t>0$.)







        share|cite|improve this answer












        share|cite|improve this answer



        share|cite|improve this answer










        answered Nov 21 '18 at 10:03









        p4sch

        4,760217




        4,760217






























            draft saved

            draft discarded




















































            Thanks for contributing an answer to Mathematics Stack Exchange!


            • Please be sure to answer the question. Provide details and share your research!

            But avoid



            • Asking for help, clarification, or responding to other answers.

            • Making statements based on opinion; back them up with references or personal experience.


            Use MathJax to format equations. MathJax reference.


            To learn more, see our tips on writing great answers.





            Some of your past answers have not been well-received, and you're in danger of being blocked from answering.


            Please pay close attention to the following guidance:


            • Please be sure to answer the question. Provide details and share your research!

            But avoid



            • Asking for help, clarification, or responding to other answers.

            • Making statements based on opinion; back them up with references or personal experience.


            To learn more, see our tips on writing great answers.




            draft saved


            draft discarded














            StackExchange.ready(
            function () {
            StackExchange.openid.initPostLogin('.new-post-login', 'https%3a%2f%2fmath.stackexchange.com%2fquestions%2f3007300%2fjoint-density-of-two-vectors-of-multivariate-normal-random-variables%23new-answer', 'question_page');
            }
            );

            Post as a guest















            Required, but never shown





















































            Required, but never shown














            Required, but never shown












            Required, but never shown







            Required, but never shown

































            Required, but never shown














            Required, but never shown












            Required, but never shown







            Required, but never shown







            Popular posts from this blog

            Biblatex bibliography style without URLs when DOI exists (in Overleaf with Zotero bibliography)

            ComboBox Display Member on multiple fields

            Is it possible to collect Nectar points via Trainline?